Position and Momentum from Wavefunctions | Quantum Mechanics

แชร์
ฝัง
  • เผยแพร่เมื่อ 11 มี.ค. 2018
  • In this video, I define the expectation value of position for a wavefunction psi and use that to derive the expectation value of momentum as well as the expressions for the position and momentum operator. I then show that in general, any Classical Mechanics quantity can be determined from the wavefunction using a combination of the position and/or momentum operators.
    There's quite a bit of math involved, so if you have any questions, let me know in the comments! Stay tuned for the next video (coming soon!) in which I will use the expressions for the momentum and position operators to derive the Heisenberg Uncertainty Principle.
    Prerequisites: All the previous videos in both of these playlists (Playlist 1: • Quantum Mechanics , Playlist 2: • Quantum Mechanics: Mat... )
    2nd Postulate Video: • Commutators and Eigenv...
    Previous Video: • Introduction to Quantu...
    Lecture Notes: drive.google.com/open?id=1ppT...
    Patreon: www.patreon.com/user?u=4354534
    Twitter: / facultyofkhan
    Special thanks to my Patrons for supporting me at the $5 level or higher:
    - Jose Lockhart
    - Yuan Gao
    - Justin Hill
    - Marcin Maciejewski
    - Jacob Soares
    - Yenyo Pal
    - Chi
    - Lisa Bouchard

ความคิดเห็น • 56

  • @_DD_15
    @_DD_15 6 ปีที่แล้ว +17

    Mate please hurry up with these videos, they are absolutely amazing!! Much appreciated, somehow the way you explain is the same way I learn things. Because I grasp everything in 1second from your lessons. Thanks a lot! I like the way you teach stuff, the detailed mathematical description. Simply perfect!

  • @julianandresflorezsolano1745
    @julianandresflorezsolano1745 3 ปีที่แล้ว +1

    This is 10 times better than the lectures my professor gives

  • @medtabernoust3869
    @medtabernoust3869 2 ปีที่แล้ว +3

    u are the best i swear u are the kind of teacher that i like, u give since to everything thank u so so so so much for those amazing video

  • @giuseppeguap7250
    @giuseppeguap7250 5 ปีที่แล้ว +2

    Bless your youtube soul

  • @kipropcollins4220
    @kipropcollins4220 ปีที่แล้ว

    this got to be the best lesson i have ever attended on QM

  • @Hevander75
    @Hevander75 6 ปีที่แล้ว +2

    thank you for making these videos

  • @aclearlight
    @aclearlight 3 ปีที่แล้ว

    Nicely done!

  • @youssefbourais534
    @youssefbourais534 5 ปีที่แล้ว +1

    Good job man ...thanx

  • @manueljenkin95
    @manueljenkin95 3 ปีที่แล้ว

    Thank you very much!

  • @reinia127
    @reinia127 5 ปีที่แล้ว

    thank you so much!!!!

  • @anmolperson1441
    @anmolperson1441 3 ปีที่แล้ว

    Thank you sir ❤️

  • @sanghabalkhande3916
    @sanghabalkhande3916 2 ปีที่แล้ว

    Thank you sir 👍

  • @munish6957
    @munish6957 3 ปีที่แล้ว +2

    Your videos are amazing... can you plz elaborate or make a video on 'how you create these videos', i mean what all hardware n software needed...

  • @samybrahimi2045
    @samybrahimi2045 2 ปีที่แล้ว

    Thanks for helping us to better understanding of quantum mechanics. Can you show us how to do for resolving the schrodinger equation for Hydrogene atom and for other atoms having several electrons with perturbation theory. Thanks again.

  • @nikitasharma3869
    @nikitasharma3869 2 ปีที่แล้ว

    Thank you sir g

  • @lutfiandre4371
    @lutfiandre4371 4 ปีที่แล้ว

    would like you to share what kind of app you used to demonstrated the explanation ?

  • @beehappy5677
    @beehappy5677 3 ปีที่แล้ว

    6.38 Doesnt the boundary limit apply to x in the first part of the solution of tbe integral? Then, doesnt it produce a infinityxcero indetermination??

  • @bikashbora5801
    @bikashbora5801 5 ปีที่แล้ว

    Please make an video on Schrödinger and Heisenberg interaction pictures

  • @ostensiblyquerulous
    @ostensiblyquerulous 6 ปีที่แล้ว +2

    Could you discuss the collapsing of the probability distribution to a delta function in a future video?

    • @FacultyofKhan
      @FacultyofKhan  6 ปีที่แล้ว

      I discussed it briefly here (at about 5:03): th-cam.com/video/kUm4q0UIpio/w-d-xo.html&l=

  • @yashupadhyay50
    @yashupadhyay50 6 ปีที่แล้ว

    sir please give lecture on step potential and particle in one dimensional box

  • @novocaine212
    @novocaine212 4 ปีที่แล้ว

    Can momentum take a complex value? For position, Ψ*(x)Ψ(x) is pure real because their phases cancel. However, for momentum, I do not see why the equation should yield a real expected value. Love the video!

  • @Sairaj922
    @Sairaj922 4 ปีที่แล้ว

    6:26 , shouldnt the limit be taken with x? But x goes to infinity even though phi goes to 0

  • @subasinidas1711
    @subasinidas1711 2 ปีที่แล้ว

    Sir, please make a video on momentum space wave function

  • @fosheimdet
    @fosheimdet 4 ปีที่แล้ว +2

    Why does the total derivative become a partial derivative when you move it inside the integral at 3:57?

    • @ztac_dex
      @ztac_dex 4 ปีที่แล้ว +1

      because psi is in terms of x and t , not just x (explanation from Griffith's QM)

    • @domzi2
      @domzi2 3 ปีที่แล้ว

      its called Leibniz integral rule, read more about it if you are interested why exactly

  • @aleksandrabobot6739
    @aleksandrabobot6739 4 ปีที่แล้ว

    At 5:16: from where can you notice that?

  • @atwellbill
    @atwellbill 2 ปีที่แล้ว

    At 8:41 in this video, you write the expectation value of momentum as the product of the mass and the (postulated) expectation value of velocity. But is this non-relativistic term for momentum applicable here? What about atoms where the electrons are moving such that expectation value of velocity

  • @HidekazuOki
    @HidekazuOki 6 ปีที่แล้ว

    Shouldn't repeated measurement of the same system yield different results based on the probability density of the wavefunction, not the same exact state each time?

    • @FacultyofKhan
      @FacultyofKhan  6 ปีที่แล้ว +1

      No, because measuring a system directly interferes with it and collapses the wavefunction (i.e. wavefunction collapse). I'd have to wait for the particle to reach steady state again before I'd get a different measurement based on the probability density of the wavefunction I originally collapsed, as I said in this video.

  • @dailydoorsteprangoli9179
    @dailydoorsteprangoli9179 2 ปีที่แล้ว

    How u create these videos? Hardware n software ...pls suggest

  • @unkownuser1042
    @unkownuser1042 2 ปีที่แล้ว

    Bro what app do you use

  • @fionachiaraviglio4302
    @fionachiaraviglio4302 4 ปีที่แล้ว

    I LOVE KAHN

  • @insignificantphysicist1295
    @insignificantphysicist1295 6 ปีที่แล้ว +1

    Can you suggest me a good book to study these basics?

    • @FacultyofKhan
      @FacultyofKhan  6 ปีที่แล้ว +1

      Griffith's is pretty nice for basic Quantum Mech. I'd start with that if I were you.

  • @giladreti
    @giladreti 6 ปีที่แล้ว +2

    I do not get it. The convergence of the improper integral does not imply the convergence of the integrated function...

    • @FacultyofKhan
      @FacultyofKhan  6 ปีที่แล้ว +2

      You're right, but pretty much every function that you'll see in Physics obeys that rule. I mentioned this in the description of the Schrodinger equation video. The only functions that have a convergent improper integral but themselves don't converge are exotic mathematical functions; we don't have to worry about those for a Physics course. Hope that helps!

    • @giladreti
      @giladreti 6 ปีที่แล้ว

      Okay, thanks!

  • @neutralgeek9492
    @neutralgeek9492 5 ปีที่แล้ว +1

    Quite fast but good explanation.

  • @shyamsunder7819
    @shyamsunder7819 5 ปีที่แล้ว

    How did derivative with respect to t turned to the derivative with respect to x

    • @FacultyofKhan
      @FacultyofKhan  5 ปีที่แล้ว +1

      I used the Schrodinger equation to make that conversion. Here is the relevant part from the previous video: th-cam.com/video/kUm4q0UIpio/w-d-xo.html
      Hope that helps!

  • @solsticetwo3476
    @solsticetwo3476 5 ปีที่แล้ว

    At 10:30 you are missing a 'X' inside the integral

    • @kovanovsky2233
      @kovanovsky2233 4 ปีที่แล้ว

      the 'X' is in parenthesis between the psi* and psi

  • @pekertimulia125
    @pekertimulia125 3 ปีที่แล้ว

    Ini adalah pembuktian bahwa semuanya memiliki solusi bagian yg paling bukan

  • @pekertimulia125
    @pekertimulia125 3 ปีที่แล้ว

    Ini bukannya sebelah mana?👍

  • @jacobvandijk6525
    @jacobvandijk6525 4 ปีที่แล้ว +1

    From 3:37 until 8:31 only hard-core math; no physics to be found there.

    • @dracowolfe305
      @dracowolfe305 3 ปีที่แล้ว

      Felt like being underwater for 5 minutes

  • @pekertimulia125
    @pekertimulia125 3 ปีที่แล้ว

    It's odd really...

  • @amedeodeamicis1957
    @amedeodeamicis1957 4 ปีที่แล้ว +1

    i didn't understand....

  • @pekertimulia125
    @pekertimulia125 3 ปีที่แล้ว

    I hate statictic..
    🙄🙄😘😭

  • @paulg444
    @paulg444 3 ปีที่แล้ว

    i disagree, the wave function as described is a function of time. You have implied that you have set up a physical system described by that wave function. If you sample from it (i.e. measure the position) and then repeat ad infinitum to aggregate an ensemble of measurements then the first moment is indeed the mean of the measurements. Speaking about wave function "collapse" in the first 2 minutes of explanation is introducing an interpretation of QM that is completely unnecessary here and it confuses the reader. Think of the wave function as described here as a likelihood function or conditional probability for the measurements system.

  • @sabuhihasanov9835
    @sabuhihasanov9835 3 ปีที่แล้ว

    Prezident niye eshitmir ki, elbette eshidir. Prezidentin her aile üzvünün bir biznesi var. O hele öz ailesinden bashlasin.... Bu veziyyete getiren mehz prezident ve onun etrafidir... Besdirin da prezidente müraciet etdiniz..

  • @arhaankhan6714
    @arhaankhan6714 2 ปีที่แล้ว

    I do not get it. The convergence of the improper integral does not imply the convergence of the integrated function...